Jednolity plik PDF różnicy dwóch rv


9

Czy możliwe jest, aby PDF różnicy dwóch iid rv wyglądał jak prostokąt (zamiast, powiedzmy, trójkąta, który otrzymujemy, jeśli rv zostaną pobrane z rozkładu jednolitego).

tzn. czy jest możliwe, aby PDF f jk (dla dwóch iid rv pobranych z jakiejś dystrybucji) miał f (x) = 0,5 dla wszystkich -1 <x <1?

Nie ma żadnych ograniczeń w dystrybucji, z której bierzemy j i k, z wyjątkiem tego, że min wynosi -1, a maksimum wynosi 1.

Po eksperymentach myślę, że może to być niemożliwe.


Różnica między dwoma rozkładami równomiernymi jest rozkładem trójkątnym, więc jeśli zapytasz, czy możliwe jest uzyskanie jednorodności różnicy między mundurami iid, odpowiedź brzmi: nie.
Tim

To samo pytanie zadane tutaj: math.stackexchange.com/questions/2048939/... jak dotąd bez odpowiedzi!
kjetil b halvorsen

Rzeczywiście trudno byłoby uniknąć realizacji na zewnątrz [1,1] gdy oboje j i kmieć masę prawdopodobieństwa zbliżoną do tych punktów końcowych.
Christoph Hanck

2
To niemożliwe. Moim wspomnieniem jest to (w nieco innej formie) już gdzieś na stronie. Zobaczę, czy uda mi się go zlokalizować
Glen_b

1
@Glen_b Być może przywołujesz stats.stackexchange.com/questions/125360/… . To nie jest całkiem duplikat, chociaż, z powodu różnicyXY zmiennych iid, choć wyrażalne jako suma X+(Y),może obejmować sumę zmiennych o nieidentycznych rozkładach. Wierzę, że trywialna modyfikacja mojego rozwiązania rozwiąże tę różnicę; Wygląda na to, że rozwiązanie Silverfish stosuje się bezpośrednio, prawie bez modyfikacji, ale najpierw trzeba usunąć wiele obcych materiałów, aby to zobaczyć.
whuber

Odpowiedzi:


10

Twierdzenie: Brak rozkładuDist dla którego ABU(1,1) kiedy A,BIID Dist.


Dowód: rozważ dwie zmienne losoweA,BIID Dist o wspólnej charakterystycznej funkcji φ. Oznaczając ich różnicę przezD=Ab. Charakterystyczną funkcją różnicy jest:

φD(t)=E(exp(itD))=E(exp(it(AB)))=E(exp(itA))E(exp(itB))=φ(t)φ(t)=φ(t)φ(t)¯=|φ(t)|2.

(Czwarty wiersz tej pracy wynika z faktu, że charakterystyczną funkcją jest hermitian .) A teraz, biorącDU(1,1) daje konkretny formularz dla φD, który jest:

φD(t)=E(exp(itD))=Rexp(itr)fD(r)dr=1211exp(itr)dr=12[exp(itr)it]r=1r=1=12exp(it)exp(it)it=12(cos(t)+isin(t))(cos(t)+isin(t))it=12(cos(t)+isin(t))(cos(t)isin(t))it=122isin(t)it=sin(t)t=sinc(t).

gdzie ta ostatnia jest (nienormalizowaną) funkcją sinc . Dlatego, aby spełnić wymagania dlaDist, potrzebujemy charakterystycznej funkcji φ z kwadratową normą podaną przez:

|φ(t)|2=φD(t)=sinc(t).

Lewa strona tego równania jest kwadratową normą i dlatego jest nieujemna, podczas gdy prawa strona jest funkcją ujemną w różnych miejscach. Dlatego nie ma rozwiązania tego równania, a zatem nie ma funkcji charakterystycznej spełniającej wymagania dla rozkładu. (Porada dla Fabiana za zwrócenie na to uwagi w powiązanym pytaniu na Mathematics.SE .) Stąd nie ma rozkładu z wymaganiami twierdzenia.


3

Jest to stanowisko inżyniera elektryka w tej kwestii, z punktu widzenia bardziej odpowiedniego dla dsp.SE niż stats.SE, ale bez względu na to.

Przypuszczam, że X i Yciągłymi zmiennymi losowymi ze wspólnym plikiem pdff(x). A następnie, jeśliZ oznacza XYmamy to

fZ(z)=f(x)f(x+z) dx.
Mówi nam o tym nierówność Cauchy'ego-Schwarza fZ(z) ma maksimum przy z=0. W rzeczywistości od tego czasufZ jest właściwie funkcją „autokorelacji” fuważany za „sygnał”, musi mieć unikalne maksimum naz=0 a zatem Z nie może być równomiernie rozłożone, jak jest to pożądane. Alternatywnie, jeślifZ były rzeczywiście jednolitą gęstością (pamiętaj, że jest to również funkcja autokorelacji), a następnie „gęstość widmowa mocy” fZ(uważany za sygnał) byłby funkcją sinusa, a zatem nie nieujemną funkcją, ponieważ muszą być wszystkie gęstości widmowe mocy. Ergo, założenie, żefZ równomierna gęstość prowadzi do sprzeczności, a zatem założenie musi być fałszywe.

Twierdzenie, że fZU[1,1] jest oczywiście nieważny, gdy wspólna dystrybucja X i Yzawiera atomy, ponieważ w takim przypadku rozkładZbędzie również zawierać atomy. Podejrzewam, że ograniczenie toX i Ymieć plik pdf, który można usunąć, a dowód czysto teoretyczny skonstruowany dla ogólnego przypadku, gdyX i Y niekoniecznie cieszą się pdf (ale ich różnica robi).


1
Częściowo wydaje mi się to niewłaściwe. Charakterystyczna funkcjaU(1,1)dystrybucja jestsincfunkcja, tak wyraźnie, że dopuszcza się rodzaj transformacji Fouriera. Wydaje mi się, że twoja logika prowadzi do udowodnienia zbyt wiele - wydaje się, że nie tylkoZnie może być jednolity, ale to, że rozkład jednolity w ogóle nie istnieje. Czy źle zrozumiałem?
Ben - Przywróć Monikę

1
Czy funkcja charakterystyczna dla U[1,1]istnieje nie jest problemem; to istnieje. Plik pdf zZjest funkcją autokorelacji . Cóż, widmowa mocy gęstość od jakiejkolwiek funkcji autokorelacji musi być nieujemną funkcją. Więc założenie, żefZU[1,1]prowadzi do gęstości widmowej mocy, która jest funkcją sinusa (która przyjmuje zarówno wartości dodatnie, jak i ujemne). Ponieważ nie jest to poprawna gęstość widmowa mocy (pamiętaj o tymfZ jest również funkcją autokorelacji), przy założeniu, że fZU[1,1]musi być fałszywe.
Dilip Sarwate,
Korzystając z naszej strony potwierdzasz, że przeczytałeś(-aś) i rozumiesz nasze zasady używania plików cookie i zasady ochrony prywatności.
Licensed under cc by-sa 3.0 with attribution required.